Difference between revisions of "2023 AMC 12A Problems/Problem 13"
m (→Video Solution ⚡️ Under 3 min ⚡️) |
(→Solution 6 (Cheese 2)) |
||
(12 intermediate revisions by 8 users not shown) | |||
Line 3: | Line 3: | ||
==Problem== | ==Problem== | ||
− | In a table tennis tournament every participant played every other participant exactly once. Although there were twice as many right-handed players as left-handed players, the number of games won by left-handed players was <math>40\%</math> more than the number of games won by right-handed players. (There were no ties and no ambidextrous players.) What is the total number of games played? | + | In a table tennis tournament, every participant played every other participant exactly once. Although there were twice as many right-handed players as left-handed players, the number of games won by left-handed players was <math>40\%</math> more than the number of games won by right-handed players. (There were no ties and no ambidextrous players.) What is the total number of games played? |
<math>\textbf{(A) }15\qquad\textbf{(B) }36\qquad\textbf{(C) }45\qquad\textbf{(D) }48\qquad\textbf{(E) }66</math> | <math>\textbf{(A) }15\qquad\textbf{(B) }36\qquad\textbf{(C) }45\qquad\textbf{(D) }48\qquad\textbf{(E) }66</math> | ||
==Solution 1== | ==Solution 1== | ||
− | We know that the total amount of games must be the sum of games won by left and right handed players. Then, we can write <math>g = l + r</math>, and since <math>l = 1.4r</math>, <math>g = 2.4r</math>. Given that <math>r</math> and <math>g</math> are both integers, <math>g/2.4</math> also must be an integer. From here we can see that <math>g</math> must be divisible by 12, leaving only answers B and D. Now we know the formula for how many games are played in this tournament is <math>n(n-1)/2</math>, the sum of the first <math>n-1</math> | + | We know that the total amount of games must be the sum of games won by left and right handed players. Then, we can write <math>g = l + r</math>, and since <math>l = 1.4r</math>, <math>g = 2.4r</math>. Given that <math>r</math> and <math>g</math> are both integers, <math>g/2.4</math> also must be an integer. From here we can see that <math>g</math> must be divisible by 12, leaving only answers B and D. Now we know the formula for how many games are played in this tournament is <math>n(n-1)/2</math>, the sum of the first <math>n-1</math> numbers. Now, setting 36 and 48 equal to the equation will show that two consecutive numbers must have a product of 72 or 96. Clearly, <math>72=8*9</math>, so the answer is <math>\boxed{\textbf{(B) }36}</math>. |
~~ Antifreeze5420 | ~~ Antifreeze5420 | ||
Line 14: | Line 14: | ||
==Solution 2== | ==Solution 2== | ||
First, we know that every player played every other player, so there's a total of <math>\dbinom{n}{2}</math> games since each pair of players forms a bijection to a game. Therefore, that rules out D. Also, if we assume the right-handed players won a total of <math>x</math> games, the left-handed players must have won a total of <math>\dfrac{7}{5}x</math> games, meaning that the total number of games played was <math>\dfrac{12}{5}x</math>. Thus, the total number of games must be divisible by <math>12</math>. Therefore leaving only answer choices B and D. Since answer choice D doesn't satisfy the first condition, the only answer that satisfies both conditions is <math>\boxed{\textbf{(B) }36}</math> | First, we know that every player played every other player, so there's a total of <math>\dbinom{n}{2}</math> games since each pair of players forms a bijection to a game. Therefore, that rules out D. Also, if we assume the right-handed players won a total of <math>x</math> games, the left-handed players must have won a total of <math>\dfrac{7}{5}x</math> games, meaning that the total number of games played was <math>\dfrac{12}{5}x</math>. Thus, the total number of games must be divisible by <math>12</math>. Therefore leaving only answer choices B and D. Since answer choice D doesn't satisfy the first condition, the only answer that satisfies both conditions is <math>\boxed{\textbf{(B) }36}</math> | ||
+ | |||
+ | |||
+ | |||
+ | A simpler way is to know that the result must be a number in the form <math>\dbinom{n}{2}</math> games. This eliminates D. Then write a expression that equals the answer, 1.4x+x=n games total. Only 48 and 36 satisfy so 36 is the answer which is <math>\boxed{\textbf{(B) }36}</math> | ||
+ | |||
+ | ~breakingbread | ||
==Solution 3== | ==Solution 3== | ||
Line 41: | Line 47: | ||
~MathFun1000 | ~MathFun1000 | ||
+ | |||
+ | ==Solution 6 (Cheese 2)== | ||
+ | |||
+ | Call the number of games x, the number of games won by right handed player R, and by left handed players L. L+R=x and L=1.4 R. Therefore, x/2.4 must be an integer, which leaves only 36 and 48. Then, note x must be half the product of two consecutive numbers (the number of possible combinations of two people) and we can eliminate 48. | ||
+ | |||
+ | ~meikh_neiht | ||
+ | |||
+ | |||
+ | How is this cheese... it is literally the first solution but scaled down | ||
+ | |||
+ | -Shreyansh | ||
+ | |||
+ | ==Solution 7== | ||
+ | |||
+ | Let the number of people be <math>n</math>. This means that there will be <math>\frac{n(n-1)}{2}</math> games played. We can now eliminate <math>\textbf{(D)}</math> because it's not a triangular number. The problem states that there are twice as many right handed people as left handed people with no ambidextrous people. Based on this, we can say that the number of people (or <math>n</math>) has to be divisible by 3. Thus, the only answer choice that fits the conditions above is answer <math>\boxed{\textbf{(B) }36}</math>. | ||
+ | |||
+ | |||
+ | ~ROGER8432V3 | ||
+ | |||
+ | ==Video Solution by Little Fermat== | ||
+ | https://youtu.be/h2Pf2hvF1wE?si=W4Ad9Bm3vhcTBB4G&t=3440 | ||
+ | ~little-fermat | ||
+ | ==Video Solution by Math-X == | ||
+ | https://youtu.be/GP-DYudh5qU?si=DCXVk-iVlqWT-6bS&t=4158 | ||
+ | ~Math-X | ||
== Video Solution by Power Solve == | == Video Solution by Power Solve == | ||
Line 65: | Line 96: | ||
~Steven Chen (Professor Chen Education Palace, www.professorchenedu.com) | ~Steven Chen (Professor Chen Education Palace, www.professorchenedu.com) | ||
− | |||
− | |||
− | |||
− | |||
==Video Solution by SpreadTheMathLove== | ==Video Solution by SpreadTheMathLove== |
Latest revision as of 10:12, 23 September 2025
- The following problem is from both the 2023 AMC 10A #16 and 2023 AMC 12A #13, so both problems redirect to this page.
Contents
- 1 Problem
- 2 Solution 1
- 3 Solution 2
- 4 Solution 3
- 5 Solution 4
- 6 Solution 5 (🧀Cheese🧀)
- 7 Solution 6 (Cheese 2)
- 8 Solution 7
- 9 Video Solution by Little Fermat
- 10 Video Solution by Math-X
- 11 Video Solution by Power Solve
- 12 Video Solution ⚡️ Under 2 min ⚡️
- 13 Video Solution 1 by OmegaLearn
- 14 Video Solution by CosineMethod
- 15 Video Solution 2 by TheBeautyofMath
- 16 Video Solution
- 17 Video Solution by SpreadTheMathLove
- 18 See Also
Problem
In a table tennis tournament, every participant played every other participant exactly once. Although there were twice as many right-handed players as left-handed players, the number of games won by left-handed players was more than the number of games won by right-handed players. (There were no ties and no ambidextrous players.) What is the total number of games played?
Solution 1
We know that the total amount of games must be the sum of games won by left and right handed players. Then, we can write , and since
,
. Given that
and
are both integers,
also must be an integer. From here we can see that
must be divisible by 12, leaving only answers B and D. Now we know the formula for how many games are played in this tournament is
, the sum of the first
numbers. Now, setting 36 and 48 equal to the equation will show that two consecutive numbers must have a product of 72 or 96. Clearly,
, so the answer is
.
~~ Antifreeze5420
Solution 2
First, we know that every player played every other player, so there's a total of games since each pair of players forms a bijection to a game. Therefore, that rules out D. Also, if we assume the right-handed players won a total of
games, the left-handed players must have won a total of
games, meaning that the total number of games played was
. Thus, the total number of games must be divisible by
. Therefore leaving only answer choices B and D. Since answer choice D doesn't satisfy the first condition, the only answer that satisfies both conditions is
A simpler way is to know that the result must be a number in the form games. This eliminates D. Then write a expression that equals the answer, 1.4x+x=n games total. Only 48 and 36 satisfy so 36 is the answer which is
~breakingbread
Solution 3
Let be the amount of games the right-handed won. Since the left-handed won
games, the total number of games played can be expressed as
, or
, meaning that the answer is divisible by 12. This brings us down to two answer choices,
and
.
We note that the answer is some number
choose
. This means the answer is in the form
. Since answer choice D gives
, and
has no integer solutions, we know that
is the only possible choice.
Solution 4
Here is the rigid way to prove that is the only answer. Let the number of left-handed players be
, so the number of right-handed players is
. The number of games won by the left-handed players comes in two ways:
- The games played by two left-left pairs, which is
, and
- The games played by left-right pairs, which we'll call
.
Note that which is the total number of games played by left-right pairs. Using the same logic for right-right pairs and right-left pairs, we have that
which gives
We know that
, applying that becomes
(We can safely divide by
because it must be positive). So the total number of players
can only be
,
, and
.
Since the total number of games is
times a non-negative integer number of games won by righties,
must be a multiple of
. Among
, only
satisfies this condition, so the total number of games is
~ggao5uiuc, oinava, yingkai_0_ (minor edits)
Solution 5 (🧀Cheese🧀)
If there are players, the total number of games played must be
, so it has to be a triangular number. The ratio of games won by left-handed to right-handed players is
, so the number of games played must also be divisible by
. Finally, we notice that only
satisfies both of these conditions.
~MathFun1000
Solution 6 (Cheese 2)
Call the number of games x, the number of games won by right handed player R, and by left handed players L. L+R=x and L=1.4 R. Therefore, x/2.4 must be an integer, which leaves only 36 and 48. Then, note x must be half the product of two consecutive numbers (the number of possible combinations of two people) and we can eliminate 48.
~meikh_neiht
How is this cheese... it is literally the first solution but scaled down
-Shreyansh
Solution 7
Let the number of people be . This means that there will be
games played. We can now eliminate
because it's not a triangular number. The problem states that there are twice as many right handed people as left handed people with no ambidextrous people. Based on this, we can say that the number of people (or
) has to be divisible by 3. Thus, the only answer choice that fits the conditions above is answer
.
~ROGER8432V3
Video Solution by Little Fermat
https://youtu.be/h2Pf2hvF1wE?si=W4Ad9Bm3vhcTBB4G&t=3440 ~little-fermat
Video Solution by Math-X
https://youtu.be/GP-DYudh5qU?si=DCXVk-iVlqWT-6bS&t=4158 ~Math-X
Video Solution by Power Solve
Video Solution ⚡️ Under 2 min ⚡️
~Education, the Study of Everything
Video Solution 1 by OmegaLearn
Video Solution by CosineMethod
https://www.youtube.com/watch?v=N-eZMOv_ZjY
Video Solution 2 by TheBeautyofMath
https://www.youtube.com/watch?v=sLtsF1k9Fx8&t=227s
Video Solution
~Steven Chen (Professor Chen Education Palace, www.professorchenedu.com)
Video Solution by SpreadTheMathLove
https://www.youtube.com/watch?v=sypOvNiR3sw
See Also
2023 AMC 10A (Problems • Answer Key • Resources) | ||
Preceded by Problem 15 |
Followed by Problem 17 | |
1 • 2 • 3 • 4 • 5 • 6 • 7 • 8 • 9 • 10 • 11 • 12 • 13 • 14 • 15 • 16 • 17 • 18 • 19 • 20 • 21 • 22 • 23 • 24 • 25 | ||
All AMC 10 Problems and Solutions |
2023 AMC 12A (Problems • Answer Key • Resources) | |
Preceded by Problem 12 |
Followed by Problem 14 |
1 • 2 • 3 • 4 • 5 • 6 • 7 • 8 • 9 • 10 • 11 • 12 • 13 • 14 • 15 • 16 • 17 • 18 • 19 • 20 • 21 • 22 • 23 • 24 • 25 | |
All AMC 12 Problems and Solutions |
These problems are copyrighted © by the Mathematical Association of America, as part of the American Mathematics Competitions.